Bullet conservation of momentum problem

In summary, the conversation discusses finding the initial speed of a bullet fired into a ballistic pendulum, given the speed of the bullet after exiting the block and the maximum height the block rises to. The solution involves using the conservation of momentum and mechanical energy, but the initial velocity is calculated incorrectly due to rounding before the final step. The correct initial velocity is 529 m/s.
  • #1
BrainMan
279
2

Homework Statement


A 7-g bullet is fired into a 1.5-kg ballistic pendulum. The bullet emerges from the block with a speed of 200 m/s, and the block rises to a maximum height of 12 cm. Find the initial speed of the bullet.

Homework Equations


conservation of momentum

The Attempt at a Solution


First I tried to find the initial momentum of the bullet as if it did't penetrate the block
so .007vi = (1.507)vf
because I have two unknowns I used the conservation of mechanical energy to solve for vf
1/2(1.507)(vf2) = (1.507)(9.8)(.12)
vf = 1.54 m/s
then plug that into find the initial velocity
vi = 435.29 m/s
initial momentum = 3.05

Then I tried to find the momentum of the bullet exiting the block and add it to the initial momentum to get the total initial momentum.
200(.007) = 1.4
1.4 + 3.05 = 4.45
and finally I divided that by the mass to get the initial velocity 635.75 m/s. The correct answer is 529 m/s.
 
Physics news on Phys.org
  • #2
BrainMan said:
vi = 435.29 m/s

How did you get this number?
BrainMan said:
0.007vi = (1.507)vf
...
vf = 1.54 m/s

From this? (If yes, then double check your numbers.)
 
  • #3
Other than that, looks good!

P.S.
You should not round your numbers until the very end. (Try to solve the problem with just algebra and then plug in the numbers at the end.) The reason is that your answer will be slightly off.

(In this problem and another you've recently posted, you rounded before the end which caused your answers to be off by about half a percent, which isn't much, but in some problems rounding before the end will cause you to be off by more than half a percent, and in other problems half a percent could be a meaningful error.)
 
  • Like
Likes 1 person
  • #4
Nathanael said:
Other than that, looks good!

P.S.
You should not round your numbers until the very end. (Try to solve the problem with just algebra and then plug in the numbers at the end.) The reason is that your answer will be slightly off.

(In this problem and another you've recently posted, you rounded before the end which caused your answers to be off by about half a percent, which isn't much, but in some problems rounding before the end will cause you to be off by more than half a percent, and in other problems half a percent could be a meaningful error.)

OK I see what I did! Thanks!
 
  • #5


Your approach of using conservation of momentum and energy is correct. However, there are a few errors in your calculation.

1. In your first attempt, you used the wrong mass for the block. The mass of the block is given as 1.5 kg, not 1.507 kg.

2. In your second attempt, you added the momentum of the bullet after exiting the block to the initial momentum. This is incorrect because the bullet is no longer in contact with the block and therefore does not contribute to the momentum of the system.

3. The final calculation for the initial velocity in your second attempt is incorrect. The correct calculation is (4.45 m/s) / (0.007 kg) = 635.71 m/s, not 635.75 m/s.

To get the correct answer of 529 m/s, you can use the following steps:

1. Calculate the initial momentum of the bullet using the given information: (0.007 kg)(vi) = (1.5 kg)(0 m/s) + (0.007 kg)(200 m/s)

2. Use conservation of energy to find the final velocity of the block: (1/2)(1.5 kg)(vf^2) = (1.5 kg)(9.8 m/s^2)(0.12 m)

3. Use conservation of momentum to find the final velocity of the bullet: (0.007 kg)(vi) = (1.5 kg)(vf)

4. Substitute the final velocity of the bullet into the equation from step 1 to solve for vi.

5. The final answer should be vi = 529 m/s.
 

1. What is the conservation of momentum principle?

The conservation of momentum principle states that in a closed system, the total momentum of all objects involved remains constant before and after a collision or interaction.

2. How is the conservation of momentum principle applied to bullet problems?

In bullet conservation of momentum problems, the total momentum of the bullet before it is fired is equal to the total momentum of the bullet and gun after it is fired. This principle can be used to solve for unknown values such as the velocity of the bullet or the recoil velocity of the gun.

3. What factors affect the conservation of momentum in bullet problems?

The mass and velocity of the bullet, as well as the mass and velocity of the gun, are the main factors that affect the conservation of momentum in bullet problems. The angle of the bullet's trajectory and any external forces, such as air resistance, may also play a role.

4. Is the conservation of momentum principle always accurate in bullet problems?

In ideal conditions, the conservation of momentum principle is accurate in bullet problems. However, factors such as friction, air resistance, and deformations of the bullet or gun may cause some deviation from the principle.

5. How is the conservation of momentum principle related to Newton's third law of motion?

Newtons's third law of motion states that for every action, there is an equal and opposite reaction. In bullet problems, the force exerted on the bullet by the gun is equal and opposite to the force exerted on the gun by the bullet, resulting in the conservation of momentum.

Similar threads

  • Introductory Physics Homework Help
Replies
10
Views
2K
  • Introductory Physics Homework Help
Replies
21
Views
1K
  • Introductory Physics Homework Help
Replies
6
Views
2K
  • Introductory Physics Homework Help
Replies
7
Views
414
  • Introductory Physics Homework Help
Replies
19
Views
1K
  • Introductory Physics Homework Help
Replies
2
Views
1K
  • Introductory Physics Homework Help
Replies
7
Views
213
  • Introductory Physics Homework Help
Replies
31
Views
3K
  • Introductory Physics Homework Help
Replies
2
Views
1K
  • Introductory Physics Homework Help
Replies
2
Views
753
Back
Top